2020-2021/TST_sti2d/06_Exponentielle_complexe/exercises.tex

271 lines
12 KiB
TeX
Raw Blame History

This file contains ambiguous Unicode characters

This file contains Unicode characters that might be confused with other characters. If you think that this is intentional, you can safely ignore this warning. Use the Escape button to reveal them.

\collectexercises{banque}
\begin{exercise}[subtitle={Multiplication entre complexe}, step={1}, origin={Création}, topics={Exponentielle complexe}, tags={Complexe}]
Soit les 4 nombres complexes sous forme algébrique
\[
z_A = 1 + \sqrt{3}i \qquad
z_B = -i + \sqrt{3} \qquad
z_C = -\frac{\sqrt{2}}{2} + \frac{\sqrt{2}}{2}i \qquad
z_D = 3\sqrt{2} + 3\sqrt{2}i
\]
\begin{enumerate}
\item Calculer le module et l'argument de ces 4 nombres complexes.
\item À partir de la forme algébrique, calculer tous les produits possibles et déterminer le module et l'argument des résultats. Vous reporterez vos résultats dans les tableaux suivants
\begin{tabular}{|c|*{6}{p{3cm}|}}
\hline
Algébrique & A & B & C & D \\
\hline
A & & & & \\
\hline
B & & & & \\
\hline
C & & & & \\
\hline
D & & & & \\
\hline
\end{tabular}
{\small
\hspace{-1cm}
\begin{tabular}{|c|*{6}{p{1.5cm}|}}
\hline
Module & A($r= \cdots$) & B($r= \cdots$) & C ($r= \cdots$)& D($r= \cdots$) \\
\hline
A ($r= \cdots$) & & & &\\
\hline
B ($r= \cdots$) & & & &\\
\hline
C ($r= \cdots$) & & & &\\
\hline
D ($r= \cdots$) & & & &\\
\hline
\end{tabular}
\hfill
\begin{tabular}{|c|*{6}{p{1.5cm}|}}
\hline
Argument & A($\theta= \cdots$) & B($\theta= \cdots$) & C($\theta= \cdots$) & D($\theta= \cdots$) \\
\hline
A ($\theta= \cdots$) & & & &\\
\hline
B ($\theta= \cdots$) & & & &\\
\hline
C ($\theta= \cdots$) & & & &\\
\hline
D ($\theta= \cdots$) & & & &\\
\hline
\end{tabular}
}
\item Compléter les phrases suivantes à partir de vos résultats
\begin{itemize}
\item Quand on multiplie 2 nombres complexes alors les modules sont \dotfill
\item Quand on multiplie 2 nombres complexes alors les arguments sont \dotfill
\end{itemize}
\end{enumerate}
\end{exercise}
\begin{solution}
\hspace{-1cm}
\begin{tabular}{|c|*{6}{p{4.25cm}|}}
\hline
& A & B & C & D \\
\hline
A & $-2 + 2 \sqrt{3} i$ & $2 \sqrt{3} + 2 i$ & $\left( - \frac{\sqrt{6}}{2} - \frac{\sqrt{2}}{2}\right) + \left(- \frac{\sqrt{6}}{2} + \frac{\sqrt{2}}{2} \right) i$ & $\left(- 3 \sqrt{6} + 3 \sqrt{2}\right) + \left(3 \sqrt{2} + 3 \sqrt{6} \right)i$\\
\hline
B & $2 \sqrt{3} + 2 i$ & $2 - 2 \sqrt{3} i$ & $\left(- \frac{\sqrt{6}}{2} + \frac{\sqrt{2}}{2}\right) + \left(\frac{\sqrt{2}}{2} + \frac{\sqrt{6}}{2}\right)i$ & $\left(3 \sqrt{2} + 3 \sqrt{6}\right) + \left( - 3 \sqrt{2} + 3 \sqrt{6} \right)i$\\
\hline
C & $\left( - \frac{\sqrt{6}}{2} - \frac{\sqrt{2}}{2}\right) + \left(- \frac{\sqrt{6}}{2} + \frac{\sqrt{2}}{2} \right) i$ & $\left(- \frac{\sqrt{6}}{2} + \frac{\sqrt{2}}{2}\right) + \left(\frac{\sqrt{2}}{2} + \frac{\sqrt{6}}{2}\right)i$ & $- i$ & $-6$\\
\hline
D & $\left(- 3 \sqrt{6} + 3 \sqrt{2}\right) + \left(3 \sqrt{2} + 3 \sqrt{6} \right)i$ & $\left( 3 \sqrt{2} + 3 \sqrt{6}\right) + \left( - 3 \sqrt{2} + 3 \sqrt{6} \right)i$& $-6$ & $36 i$\\
\hline
\end{tabular}
\bigskip
{\small
\hspace{-1cm}
\begin{tabular}{|c|*{6}{p{1.5cm}|}}
\hline
Module & A($r= 2$) & B($r= 2$) & C ($r= 1$)& D($r= 6$) \\
\hline
A ($r= 2$) & 4 & 4 & 2 & 12\\
\hline
B ($r= 2$) & 4 & 4 & &\\
\hline
C ($r= 1$) & 2 & & &\\
\hline
D ($r= 6$) & 12 & & &\\
\hline
\end{tabular}
\hfill
\begin{tabular}{|c|*{6}{p{1.5cm}|}}
\hline
Argument & A($\theta= \frac{\pi}{3}$) & B($\theta= \frac{5\pi}{6}$) & C($\theta= \frac{3\pi}{4}$) & D($\theta= \frac{\pi}{4}$) \\
\hline
A ($\theta= \frac{\pi}{3}$) & $\frac{2\pi}{3}$ & $\frac{7\pi}{6}$ & $\frac{13\pi}{12}$ & $\frac{7\pi}{12}$ \\
\hline
B ($\theta= \frac{5\pi}{6}$) & $\frac{7\pi}{6}$ & $\frac{10\pi}{6}$ & &\\
\hline
C ($\theta= \frac{3\pi}{4}$) & $\frac{13\pi}{12}$ & & &\\
\hline
D ($\theta= \frac{\pi}{4}$) & $\frac{7\pi}{12}$^& & &\\
\hline
\end{tabular}
}
\end{solution}
\begin{exercise}[subtitle={Algébrique -> Exponentielle}, step={2}, origin={Création}, topics={Exponentielle complexe}, tags={Complexe}]
Placer les nombres suivants sur le plan complexe puis mettre sous forme exponentielle.
\begin{multicols}{3}
\begin{enumerate}
\item $z_1 = 1$
\item $z_2 = -3i$
\item $z_3 = 1 + i\sqrt{3}$
\item $z_4 = \sqrt{3} + i$
\item $z_5 = 1 - i$
\item $z_6 = \frac{-1 - i\sqrt{3}}{2}$
\end{enumerate}
\end{multicols}
\end{exercise}
\begin{exercise}[subtitle={Exponentielle -> Algébrique}, step={2}, origin={Création}, topics={Exponentielle complexe}, tags={Complexe}]
Placer les nombres suivants sur le plan complexe puis mettre sous forme algébrique.
\begin{multicols}{3}
\begin{enumerate}
\item $z_1 = e^{i\pi}$
\item $z_2 = 1e^{i\frac{\pi}{4}}$
\item $z_3 = e^{-i\frac{\pi}{2}}$
\item $z_4 = 5e^{-i\frac{4\pi}{3}}$
\item $z_5 = 10e^{i\frac{2\pi}{6}}$
\item $z_6 = \frac{1}{2}e^{i\pi}$
\end{enumerate}
\end{multicols}
\end{exercise}
\begin{exercise}[subtitle={Opération avec la forme trigonométrique}, step={2}, origin={Création}, topics={Exponentielle complexe}, tags={Complexe}]
On définit les nombres complexes suivants
\[
z_1 = \frac{\sqrt{2}}{2} - i\frac{\sqrt{2}}{2} \qquad z_2 = 1 - i\sqrt{3}
\]
\begin{enumerate}
\item Déterminer la forme exponentielle des nombres complexes.
\item Effectuer les opérations suivantes et donner le résultat sous forme exponentielle.
\begin{multicols}{2}
\begin{enumerate}
\item $z_a = z_1 \times z_2$
\item $z_b = \dfrac{z_1}{z_2}$
\end{enumerate}
\end{multicols}
\item Calculer les quantités suivantes
\begin{multicols}{3}
\begin{enumerate}
\item $z_A = z_1^2$
\item $z_B = z_1^3$
\item $z_C = z_2^4$
\end{enumerate}
\end{multicols}
\item Placer le résultat de ces opérations dans un repère.
\end{enumerate}
\end{exercise}
\begin{exercise}[subtitle={Filtres}, step={3}, origin={Création}, topics={Exponentielle complexe}, tags={Complexe}]
Les résistances et les condensateurs sont des composants électroniques utilisés dans le domaine du son pour concevoir des filtres.
Placé en sortie d'un microphone, un filtre atténue plus ou moins les sons selon leur fréquence $f$, exprimée en Hertz (Hz).
Pour un filtre donné, l'atténuation d'un son se calcule à l'aide de deux nombres complexes $z_R$.
Dans tout l'exercice, on suppose que $z_R = 10$ et $z_C = - \dfrac{\np{1000}\sqrt{3}}{f}i$ , où i désigne le nombre complexe de module 1 et d'argument $\dfrac{\pi}{2}$..
\noindent
\textbf{Partie A : Effet du filtre sur un son grave}
On choisit un son grave de fréquence $f = 100$.
\begin{enumerate}
\item Montrer que $z_C = - 10\sqrt{3} i$.
\item
\begin{enumerate}
\item Déterminer la forme exponentielle de $z_C$.
\item On considère le nombre complexe $Z = z_R + z_C$. On a donc $Z = 10 - 10\sqrt{3} i$.
Déterminer la forme exponentielle de $Z$.
\item On considère le nombre complexe $z_G$ défini par : $z_G = \dfrac{z_C}{z_R + z_C}$.
Montrer que $z_G = \dfrac{\sqrt{3}}{2}\text{e}^{- i\frac{\pi}{6}}$.
\item Le module du nombre complexe $z_G$ est appelé gain du filtre.
Donner la valeur exacte du gain du filtre puis une valeur approchée au centième.
\end{enumerate}
\end{enumerate}
\noindent
\textbf{Partie B : Effet du filtre sur un son aigu }
On choisit un son aigu de fréquence $f = \np{1000}\sqrt{3}$.
\begin{enumerate}
\item Montrer que le nombre complexe $z_G$ défini par $z_G = \dfrac{z_C}{z_R + z_C}$ est égal à $\dfrac{- i}{10 - i}$.
\item Déterminer la forme algébrique de $z_G$ .
\item Calculer la valeur exacte du gain du filtre $\left|z_G\right|$ et en donner une valeur approchée au centième.
\end{enumerate}
\end{exercise}
\begin{exercise}[subtitle={Bras articulé}, step={3}, origin={Création}, topics={Exponentielle complexe}, tags={Complexe}]
Dans le plan muni d'un repère orthonormé direct \Ouv, le bras articulé dun robot, fixé au point O, est représenté par deux segments [OA] et [AB], chacun de longueur 2 unités.
Deux exemples de position du bras articulé sont donnés ci-dessous à titre indicatif.
\noindent
\begin{minipage}{0.5\linewidth}
\begin{enumerate}
\item
\begin{enumerate}
\item Tracer sur la copie un repère orthonormé \Ouv.
Placer le point A d'affixe $z_{\text A}= 2i$ puis construire l'extrémité B du bras articulé
lorsque son affixe $z_{\text B}$ a pour argument $\dfrac{\pi}{4}$.
\item Donner l'affixe du point B sous forme algébrique et sous forme exponentielle.
\end{enumerate}
\end{enumerate}
\end{minipage}
\hfill
\begin{minipage}{0.4\linewidth}
\includegraphics[scale=0.15]{./fig/bras1}
\end{minipage}
\begin{enumerate}
\setcounter{enumi}{1}
\item L'extrémité B du bras peut-elle atteindre un objet qui se trouve à une distance de $4,5$
unités du point O?
\end{enumerate}
\noindent
\begin{minipage}{0.6\linewidth}
\begin{enumerate}
\setcounter{enumi}{2}
\item Pour soulever un objet lourd dont le point d'accroche est le point C (voir figure ci-contre), il faut rigidifier l'articulation en A. On décide alors de bloquer l'angle $\left ( \vec{AO}~,~\vec{AB}\right )$ tel qu'une mesure de cet angle soit constamment égale à $\dfrac{\pi}{2}$ radians.
\begin{enumerate}
\item Déterminer la longueur OB.
\item Le point C a pour affixe $z_{\text C} = 2\sqrt{2}\e^{i\frac{\pi}{12}}$.
Justifier que l'extrémité B du bras articulé pourra atteindre le point d'accroche C de l'objet.
\item Lorsque le bras articulé saisit l'objet, les points B et C sont confondus.
Calculer la mesure de l'angle que forme alors le bras [OA] avec l'axe [O$x$).
\end{enumerate}
\end{enumerate}
\end{minipage}
\hfill
\begin{minipage}{0.35\linewidth}
\includegraphics[scale=0.25]{./fig/bras2}
\end{minipage}
\end{exercise}
\collectexercisesstop{banque}